11
$\begingroup$

Let $E$ be an ordered field and let $F$ be a real closed subfield. We say that $E$ is $F$-archimedean if for each $e\in E$ there is $x\in F$ such that $-x\le e\le x$.

Is it true that if $E$ is $F$-archimedean then every interval in $E$ contains an element in $F$? That is, is it true that for every $e<e'$ in $E$ there is an element $x\in F$ such that $e<x<e'$?


This is known if $F$ is the field of real algebraic numbers (in which case $E$ is an ordered subfield of $\mathbb{R}$), and it seems to me that it should have an easy proof in the general case. However I cannot find neither an easy proof nor a counterexample.

$\endgroup$
5
  • 2
    $\begingroup$ Have you tried $E = \mathbb{Q}((\infty)), F = \mathbb{Q}((\infty^2))$? $\endgroup$
    – user44191
    Dec 17, 2018 at 19:49
  • 1
    $\begingroup$ What is $\mathbb{Q}((\infty))$? Do you mean $\mathbb{Q}(t)$ with the order making $t$ an infinite positive element (I'm not sure this order can be extended to the Laurent series)? $\endgroup$ Dec 17, 2018 at 19:50
  • $\begingroup$ Yes, sorry, my idiocy. $\endgroup$
    – user44191
    Dec 17, 2018 at 19:51
  • $\begingroup$ No need to be self-deprecating it's a good counterexample... Uhm there might in fact be no number between $t$ and $t+1$. Can you do a counterexample where $F$ is real closed? Sorry for changing the goal posts, I should have put it from the beginning since I was really thinking about that case. $\endgroup$ Dec 17, 2018 at 19:52
  • 1
    $\begingroup$ Try $E$ the real closure of $\mathbb{Q}(x, y)$ with $x > \mathbb{Q}, y > \mathbb{Q}(x)$, and $F$ the real closure of $\mathbb{Q}(y)$? $\endgroup$
    – user44191
    Dec 17, 2018 at 20:00

3 Answers 3

11
$\begingroup$

Let $F$ be any real-closed field of uncountable cofinality. That is, every countable subset of $F$ is bounded. One can make such a field in a process of $\omega_1$-many field extensions; alternatively, the ultrapower of $\mathbb{R}$ by nonprincipal ultrafilter on $\omega$ also has uncountable cofinality.

Let $E=F^\omega/\mu$ be an ultrapower of $F$ by a nonprincipal ultrafilter $\mu$ on $\omega$. We may identify $F$ with its canonical copy in $E$ using equivalence classes of constant functions $x\mapsto [c_x]_\mu$.

Since every function from $\omega$ to $F$ is bounded by a constant function, it follows that $E$ is $F$-archimedean. But $F$ is not dense in $E$, since there are no constant functions between $[\text{id}]_\mu$ and $[\text{id}+1]_\mu$, where $\text{id}:n\mapsto n$, viewing $\omega\subset F$.

$\endgroup$
2
  • 1
    $\begingroup$ Oh well... I guess that was too good to be true. Thanks for the answer, I'll wait till tomorrow to see if other interesting answers pop up and then accept it. $\endgroup$ Dec 17, 2018 at 20:08
  • 1
    $\begingroup$ Sure thing, no problem. I liked your question. The argument I give shows that no first-order property of the ordered field (like being real-closed) can have the consequence you want, since we can always find such fields of uncountable cofinality and then take an ultrapower. $\endgroup$ Dec 17, 2018 at 20:22
14
$\begingroup$

Let $E$ be the real closure of $\mathbb{Q}(x, y) = (\mathbb{Q}(x))(y)$, with order given by $x > \mathbb{Q}$ and$y > \mathbb{Q}(x)$. In other words, positivity on $\mathbb{Q}(x, y)$ is determined first by degree in $y$, and then by degree in $x$. Then let $F$ be the real closure of $\mathbb{Q}(y)$.

First, we prove that $E$ is $F$-archimedean. Let $e \in E$. There is some $e' \in \mathbb{Q}(x, y)$ with $e' > e > -e'$. Then $e'$ has degree $n$ in $y$ for some $y$; let $f = y^{n + 1}$. By the order on $\mathbb{Q}(x, y)$, we have $f > e' > e > -e' > -f$. Therefore $E$ is $F$-archimedean.

On the other hand, clearly, there is no element of $F$ between $x$ and $x + 1$.

$\endgroup$
2
  • 1
    $\begingroup$ Thank you for your answer! Now I'm going to have a hard time deciding which one to accept :) $\endgroup$ Dec 17, 2018 at 20:35
  • 1
    $\begingroup$ I ended up accepting JDH's answer because it provides me with a "machine" to generate counterexamples to similar statements, but your simple answer was very appreciated (and possibly shows that I hadn't thought through the situation as much as I thought...) $\endgroup$ Dec 18, 2018 at 22:37
5
$\begingroup$

This is similar to user44191's answer but I want to put the emphasis on the fact that there is no reason that the cofinality of $F$ in $E$ (which is what ou call [$E$ is $F$-archimedean]) imply the density of $F$ in $E$.

Indeed, if $F$ is any non-archimedean ordered field, then the field $F(t)$ can be equipped with an order where $t$ is positive infinite but smaller than any positive infinite element of $F$. Thus $F$ is cofinal in $F(t)$ but not dense in it since no element of $F$ is close to $t$. To do so, say that a fraction $\frac{P(t)}{Q(t)}$ is positive if $P(t)$ and $Q(t)$ have the same sign, where the sign of a polynomial $R(t)$ is positive if $R(s)$ is positive for sufficiently large finite element $s$ of $F$.

This is a special case of filling a cut in an ordered field using a simple extension.

$\endgroup$

Your Answer

By clicking “Post Your Answer”, you agree to our terms of service and acknowledge you have read our privacy policy.

Not the answer you're looking for? Browse other questions tagged or ask your own question.